Expected value problem (probability)

Click For Summary
The discussion revolves around calculating the expected value E(Y1+Y2) and variance V(Y1+Y2) for discrete random variables Y1 and Y2, constrained within specific integer ranges. The user initially attempted a direct counting method to find probabilities for Y1+Y2 equaling 1, 2, and 3, leading to an expected value of 7/3. However, they express confusion about applying the provided formulas for expectation and variance, particularly for discrete variables. The conversation emphasizes that while integration is used for continuous variables, discrete expectations can be calculated as sums of probabilities multiplied by their outcomes. Ultimately, the user is encouraged to redefine the variables and apply the formulas correctly for discrete cases.
compliant
Messages
43
Reaction score
0

Homework Statement


Given Y1 and Y2 are integer values, where 0\leqY1\leq3, 0\leqY2\leq3, 1\leqY1+Y2\leq3

p(Y1, Y2) = \frac{{4 \choose y_1}{3 \choose y_2}{2 \choose {3-y_1-y_2}}}{{9 \choose 3}}

Find E(Y1+Y2) and V(Y1+Y2)

Homework Equations


E(Y1+Y2) = E1(Y1)+E2(Y2)

E1(Y1) = \int_{-{\infty}}^{\infty} y_1 f(y_1) dy_1

But that's for continuous variables, so I have no idea how to deal with discrete. Some guy tried explaining it to me, but it was just so unclear I didn't understand any of it.

Required use of these theorems:

Given U1 = \sum_{i=1}^n a_i*Y_i
E(U1) = \sum_{i=1}^n a_i*E_i (Y_i)

V(U1) = {\sum_{i=1}^n {a_i}^2*E_i (Y_i)}+2{\sum{\sum_{1{\leq{i}}<j{\leq{n}}} {a_i}^2*E_i (Y_i)}}


The Attempt at a Solution


Well, first I did it the slow counting way involving counting,
P(Y1+Y2=1)

= \frac{{4 \choose 0}{3 \choose 1}{2 \choose {2}}}{{9 \choose 3}} + \frac{{4 \choose 1}{3 \choose 0}{2 \choose {2}}}{{9 \choose 3}}

= \frac {3+4}{84}

= \frac {7}{84}

P(Y1+Y2=2)

= \frac{{4 \choose 0}{3 \choose 2}{2 \choose {1}}}{{9 \choose 3}} + \frac{{4 \choose 1}{3 \choose 1}{2 \choose {1}}}{{9 \choose 3}} + \frac{{4 \choose 2}{3 \choose 0}{2 \choose {1}}}{{9 \choose 3}}

= \frac {3(2)+4(3)(2)+6(2)}{84}

= \frac {42}{84}

P(Y1+Y2=3)

= \frac{{4 \choose 0}{3 \choose 3}{2 \choose {0}}}{{9 \choose 3}} + \frac{{4 \choose 1}{3 \choose 2}{2 \choose {0}}}{{9 \choose 3}} + \frac{{4 \choose 2}{3 \choose 1}{2 \choose {0}}}{{9 \choose 3}} + \frac{{4 \choose 3}{3 \choose 0}{2 \choose {0}}}{{9 \choose 3}}

= \frac {1+4(3)+6(3)+4)}{84}

= \frac {35}{84}


E(Y1+Y2) = P(Y1+Y2=1)+2P(Y1+Y2=2)+3P(Y1+Y2=3)

= {(1)}{\frac {7}{84}}+{(2)}{\frac {42}{84}}+{(3)}{\frac {35}{84}}

= \frac {196}{84}

= \frac {7}{3}


But I'm supposed to be using the formula I listed above. So I have no idea how to deal with that.
 
Physics news on Phys.org
well, expectation is the same for discrete as for random variables. If you think about it, integration is a really awesome way to take sums (ie riemann sums), so the expectation of discrete random variables is just the sum over all i of x(i)times the probability of x(i)

also, you need to define U(i) as a new variable Y(1)+Y(2), and do the calculation on U. then you can use your formulas
 
Question: A clock's minute hand has length 4 and its hour hand has length 3. What is the distance between the tips at the moment when it is increasing most rapidly?(Putnam Exam Question) Answer: Making assumption that both the hands moves at constant angular velocities, the answer is ## \sqrt{7} .## But don't you think this assumption is somewhat doubtful and wrong?

Similar threads

  • · Replies 7 ·
Replies
7
Views
2K
  • · Replies 6 ·
Replies
6
Views
2K
Replies
3
Views
2K
  • · Replies 2 ·
Replies
2
Views
1K
  • · Replies 5 ·
Replies
5
Views
1K
  • · Replies 2 ·
Replies
2
Views
3K
  • · Replies 9 ·
Replies
9
Views
4K
  • · Replies 2 ·
Replies
2
Views
1K
Replies
6
Views
1K
Replies
2
Views
1K